LSAT and Law School Admissions Forum

Get expert LSAT preparation and law school admissions advice from PowerScore Test Preparation.

 nrpandolfo
  • Posts: 33
  • Joined: Feb 04, 2018
|
#45319
I initially had it narrowed down to A,B, or D. The only reason is that Dr. Han is proposing something against something he actually does engage in, smoking. which is the same as Mullen who is proposing raising taxes on something he benefitted from?

Is that the right way to go about this?
 Daniel Stern
PowerScore Staff
  • PowerScore Staff
  • Posts: 81
  • Joined: Feb 07, 2018
|
#45334
Your reasoning is exactly the right way to go about this!

The flaw in the stimulus is a source attack: the only reason given by the author for discounting Mullen's proposal to tax the rich is that records show Mullen himself invested in businesses and showed large profits. Yet to say Mullen has benefited in the past from the thing he now seeks to tax tells us nothing about whether his proposal is a good idea or not.

All of the answers contain source attacks.
A has Smith proposing legislation that he might potentially benefit from, since we're told that Smith is in the category ("working parents") that would benefit from the legistlation. However, A doesn't quite have Smith proposing an imposition on something that he benefitted from in the past.

Credited response B has Han propose a ban on smoking, but we're told we shouldn't listen, because Han was a heavy smoker. This is the same structure as our stimulus argument that Mullen's proposal to tax the rich should be discounted because he is rich.

D has Timm proposing something that a member of Timm's family may benefit from; again, not quite the same as the source attack set up in the stimulus.

You're on the right track, keep it up!
Dan
 akanshalsat
  • Posts: 104
  • Joined: Dec 20, 2017
|
#48929
Hi :)

I'm still stuck on this question -- I really don't get why D is wrong --> its also a ad hominem attack, that bc timm's daughter is a part of the company, his proposal shouldnt be considered since he isnt trustworthy, and the same goes for the stimulus, since mullen has invested before, then he isnt trustworthy and his proposal shouldnt been considered?
 Adam Tyson
PowerScore Staff
  • PowerScore Staff
  • Posts: 5153
  • Joined: Apr 14, 2011
|
#49758
Check Dave's explanation in this thread, akanshalsat. As he says, EVERY answer choice contains a source argument! We have to dig deeper as a result, and find the source argument that is most similar to the one in the stimulus.

In the stimulus, we are attacking the proposal because the person advancing it is in the group of people who might stand to LOSE something as a result of it being passed. Mullen wants to tax the rich, but he is one of the people who would be targeted by the new tax, so don't believe him. Odd argument, to be sure, because you would think you would trust him even more since he is proposing something that might go against his interest. Still, that's what the author said, and so that is what we should look for - someone proposing something that will actually target them, causing them a loss rather than benefiting them.

In answer D, there are two reasons why it fails to parallel the stimulus. First, Timm's proposal does not target Timm, but Timm's daughter. That ought to be enough to eliminate it, but then it gets worse, because it doesn't attack her, causing her a loss, but benefits her by raising her salary. Two strikes against that answer, and it must be a loser.
 student987
  • Posts: 28
  • Joined: Apr 09, 2018
|
#59602
Daniel Stern wrote:However, A doesn't quite have Smith proposing an imposition on something that he benefitted from in the past.
But (B) is also not talking about the past--it just says "Dr. Han is a heavy smoker." Is there another way to cross out A?

I'm still a bit confused on how to eliminate A, C, and E. I would appreciate any further explanation!
 Robert Carroll
PowerScore Staff
  • PowerScore Staff
  • Posts: 1787
  • Joined: Dec 06, 2013
|
#61164
student,

The past is irrelevant. In answer choice (A), the proposed legislation would benefit the person. This differs from the stimulus, where the proposal would be harmful to the person. Only answer choice (B) has a situation where the proposal would be harmful, like the stimulus does.

Robert Carroll

Get the most out of your LSAT Prep Plus subscription.

Analyze and track your performance with our Testing and Analytics Package.